Proving v1+v2 is not an eigenvector of A











up vote
3
down vote

favorite












Let $lambda_1$ and $lambda_2$ be two distinct eigenvalues of an $n times n$ matrix $A$, $v_1$ and $v_2$ are the corresponding eigenvectors. Prove that $v_1 + v_2$ is not an eigenvector of $A$.



Is this how you set this up? Unsure where to begin.



$A(v_1+v_2) = Av_1 + Av_2$



$A(v_1+v_2) = lambda_1v_1 + lambda_2v_2$



...










share|cite|improve this question









New contributor




jake is a new contributor to this site. Take care in asking for clarification, commenting, and answering.
Check out our Code of Conduct.




















  • Have you already encountered a proof that with the same assumptions ($lambda_1$ and $lambda_2$ are two distinct eigenvalues of an $ntimes n$ matrix $A$, $v_1$ and $v_2$ are their eigenvectors), $v_1$ and $v_2$ are linearly independent? If so, then use that. If not, then that is the place to start.
    – Misha Lavrov
    2 days ago

















up vote
3
down vote

favorite












Let $lambda_1$ and $lambda_2$ be two distinct eigenvalues of an $n times n$ matrix $A$, $v_1$ and $v_2$ are the corresponding eigenvectors. Prove that $v_1 + v_2$ is not an eigenvector of $A$.



Is this how you set this up? Unsure where to begin.



$A(v_1+v_2) = Av_1 + Av_2$



$A(v_1+v_2) = lambda_1v_1 + lambda_2v_2$



...










share|cite|improve this question









New contributor




jake is a new contributor to this site. Take care in asking for clarification, commenting, and answering.
Check out our Code of Conduct.




















  • Have you already encountered a proof that with the same assumptions ($lambda_1$ and $lambda_2$ are two distinct eigenvalues of an $ntimes n$ matrix $A$, $v_1$ and $v_2$ are their eigenvectors), $v_1$ and $v_2$ are linearly independent? If so, then use that. If not, then that is the place to start.
    – Misha Lavrov
    2 days ago















up vote
3
down vote

favorite









up vote
3
down vote

favorite











Let $lambda_1$ and $lambda_2$ be two distinct eigenvalues of an $n times n$ matrix $A$, $v_1$ and $v_2$ are the corresponding eigenvectors. Prove that $v_1 + v_2$ is not an eigenvector of $A$.



Is this how you set this up? Unsure where to begin.



$A(v_1+v_2) = Av_1 + Av_2$



$A(v_1+v_2) = lambda_1v_1 + lambda_2v_2$



...










share|cite|improve this question









New contributor




jake is a new contributor to this site. Take care in asking for clarification, commenting, and answering.
Check out our Code of Conduct.











Let $lambda_1$ and $lambda_2$ be two distinct eigenvalues of an $n times n$ matrix $A$, $v_1$ and $v_2$ are the corresponding eigenvectors. Prove that $v_1 + v_2$ is not an eigenvector of $A$.



Is this how you set this up? Unsure where to begin.



$A(v_1+v_2) = Av_1 + Av_2$



$A(v_1+v_2) = lambda_1v_1 + lambda_2v_2$



...







linear-algebra matrices






share|cite|improve this question









New contributor




jake is a new contributor to this site. Take care in asking for clarification, commenting, and answering.
Check out our Code of Conduct.











share|cite|improve this question









New contributor




jake is a new contributor to this site. Take care in asking for clarification, commenting, and answering.
Check out our Code of Conduct.









share|cite|improve this question




share|cite|improve this question








edited 2 days ago









platty

2,093214




2,093214






New contributor




jake is a new contributor to this site. Take care in asking for clarification, commenting, and answering.
Check out our Code of Conduct.









asked 2 days ago









jake

211




211




New contributor




jake is a new contributor to this site. Take care in asking for clarification, commenting, and answering.
Check out our Code of Conduct.





New contributor





jake is a new contributor to this site. Take care in asking for clarification, commenting, and answering.
Check out our Code of Conduct.






jake is a new contributor to this site. Take care in asking for clarification, commenting, and answering.
Check out our Code of Conduct.












  • Have you already encountered a proof that with the same assumptions ($lambda_1$ and $lambda_2$ are two distinct eigenvalues of an $ntimes n$ matrix $A$, $v_1$ and $v_2$ are their eigenvectors), $v_1$ and $v_2$ are linearly independent? If so, then use that. If not, then that is the place to start.
    – Misha Lavrov
    2 days ago




















  • Have you already encountered a proof that with the same assumptions ($lambda_1$ and $lambda_2$ are two distinct eigenvalues of an $ntimes n$ matrix $A$, $v_1$ and $v_2$ are their eigenvectors), $v_1$ and $v_2$ are linearly independent? If so, then use that. If not, then that is the place to start.
    – Misha Lavrov
    2 days ago


















Have you already encountered a proof that with the same assumptions ($lambda_1$ and $lambda_2$ are two distinct eigenvalues of an $ntimes n$ matrix $A$, $v_1$ and $v_2$ are their eigenvectors), $v_1$ and $v_2$ are linearly independent? If so, then use that. If not, then that is the place to start.
– Misha Lavrov
2 days ago






Have you already encountered a proof that with the same assumptions ($lambda_1$ and $lambda_2$ are two distinct eigenvalues of an $ntimes n$ matrix $A$, $v_1$ and $v_2$ are their eigenvectors), $v_1$ and $v_2$ are linearly independent? If so, then use that. If not, then that is the place to start.
– Misha Lavrov
2 days ago












2 Answers
2






active

oldest

votes

















up vote
5
down vote













By contradiction:



If $v_1 + v_2$ is an eigenvector of A then there exists and eigenvalue $lambda$ so that $$ A(v_1 + v_2) = lambda(v_1 + v_2) = lambda v_1 + lambda v_2.$$
However since $v_1$ and $v_2$ are eigenvectors and $A$ is linear we have
$$ A(v_1 + v_2) = A(v_1) + A(v_2) = lambda_1 v_1 + lambda_2v_2.$$
Therefore
$$ lambda v_1 + lambda v_2 = lambda_1 v_1 + lambda_2v_2$$
$$ iff$$
$$ (lambda - lambda_1) v_1 + (lambda - lambda_2)v_2 = 0. $$
Since $lambda_1 neq lambda_2$, $v_1$ and $v_2$ are linearly independent so
$$ lambda - lambda_1 = 0 qquad lambda-lambda_2 = 0.$$
So $ lambda = lambda_1 = lambda_2 $ which is a contradiction.






share|cite|improve this answer




























    up vote
    3
    down vote













    What you have above is true. It may help to write $lambda_2 = lambda_1 + c$ where $c neq 0$. Then you can write $A(v_1 + v_2) = lambda_1 (v_1 + v_2) + cv_2$. From here, you should be able to argue that $v_2$ is not parallel to $v_1 + v_2$, based on the assumption that $lambda_1 neq lambda_2$, so $v_1 + v_2$ cannot be an eigenvector of $A$.






    share|cite|improve this answer





















      Your Answer





      StackExchange.ifUsing("editor", function () {
      return StackExchange.using("mathjaxEditing", function () {
      StackExchange.MarkdownEditor.creationCallbacks.add(function (editor, postfix) {
      StackExchange.mathjaxEditing.prepareWmdForMathJax(editor, postfix, [["$", "$"], ["\\(","\\)"]]);
      });
      });
      }, "mathjax-editing");

      StackExchange.ready(function() {
      var channelOptions = {
      tags: "".split(" "),
      id: "69"
      };
      initTagRenderer("".split(" "), "".split(" "), channelOptions);

      StackExchange.using("externalEditor", function() {
      // Have to fire editor after snippets, if snippets enabled
      if (StackExchange.settings.snippets.snippetsEnabled) {
      StackExchange.using("snippets", function() {
      createEditor();
      });
      }
      else {
      createEditor();
      }
      });

      function createEditor() {
      StackExchange.prepareEditor({
      heartbeatType: 'answer',
      convertImagesToLinks: true,
      noModals: true,
      showLowRepImageUploadWarning: true,
      reputationToPostImages: 10,
      bindNavPrevention: true,
      postfix: "",
      imageUploader: {
      brandingHtml: "Powered by u003ca class="icon-imgur-white" href="https://imgur.com/"u003eu003c/au003e",
      contentPolicyHtml: "User contributions licensed under u003ca href="https://creativecommons.org/licenses/by-sa/3.0/"u003ecc by-sa 3.0 with attribution requiredu003c/au003e u003ca href="https://stackoverflow.com/legal/content-policy"u003e(content policy)u003c/au003e",
      allowUrls: true
      },
      noCode: true, onDemand: true,
      discardSelector: ".discard-answer"
      ,immediatelyShowMarkdownHelp:true
      });


      }
      });






      jake is a new contributor. Be nice, and check out our Code of Conduct.










      draft saved

      draft discarded


















      StackExchange.ready(
      function () {
      StackExchange.openid.initPostLogin('.new-post-login', 'https%3a%2f%2fmath.stackexchange.com%2fquestions%2f3016461%2fproving-v1v2-is-not-an-eigenvector-of-a%23new-answer', 'question_page');
      }
      );

      Post as a guest















      Required, but never shown

























      2 Answers
      2






      active

      oldest

      votes








      2 Answers
      2






      active

      oldest

      votes









      active

      oldest

      votes






      active

      oldest

      votes








      up vote
      5
      down vote













      By contradiction:



      If $v_1 + v_2$ is an eigenvector of A then there exists and eigenvalue $lambda$ so that $$ A(v_1 + v_2) = lambda(v_1 + v_2) = lambda v_1 + lambda v_2.$$
      However since $v_1$ and $v_2$ are eigenvectors and $A$ is linear we have
      $$ A(v_1 + v_2) = A(v_1) + A(v_2) = lambda_1 v_1 + lambda_2v_2.$$
      Therefore
      $$ lambda v_1 + lambda v_2 = lambda_1 v_1 + lambda_2v_2$$
      $$ iff$$
      $$ (lambda - lambda_1) v_1 + (lambda - lambda_2)v_2 = 0. $$
      Since $lambda_1 neq lambda_2$, $v_1$ and $v_2$ are linearly independent so
      $$ lambda - lambda_1 = 0 qquad lambda-lambda_2 = 0.$$
      So $ lambda = lambda_1 = lambda_2 $ which is a contradiction.






      share|cite|improve this answer

























        up vote
        5
        down vote













        By contradiction:



        If $v_1 + v_2$ is an eigenvector of A then there exists and eigenvalue $lambda$ so that $$ A(v_1 + v_2) = lambda(v_1 + v_2) = lambda v_1 + lambda v_2.$$
        However since $v_1$ and $v_2$ are eigenvectors and $A$ is linear we have
        $$ A(v_1 + v_2) = A(v_1) + A(v_2) = lambda_1 v_1 + lambda_2v_2.$$
        Therefore
        $$ lambda v_1 + lambda v_2 = lambda_1 v_1 + lambda_2v_2$$
        $$ iff$$
        $$ (lambda - lambda_1) v_1 + (lambda - lambda_2)v_2 = 0. $$
        Since $lambda_1 neq lambda_2$, $v_1$ and $v_2$ are linearly independent so
        $$ lambda - lambda_1 = 0 qquad lambda-lambda_2 = 0.$$
        So $ lambda = lambda_1 = lambda_2 $ which is a contradiction.






        share|cite|improve this answer























          up vote
          5
          down vote










          up vote
          5
          down vote









          By contradiction:



          If $v_1 + v_2$ is an eigenvector of A then there exists and eigenvalue $lambda$ so that $$ A(v_1 + v_2) = lambda(v_1 + v_2) = lambda v_1 + lambda v_2.$$
          However since $v_1$ and $v_2$ are eigenvectors and $A$ is linear we have
          $$ A(v_1 + v_2) = A(v_1) + A(v_2) = lambda_1 v_1 + lambda_2v_2.$$
          Therefore
          $$ lambda v_1 + lambda v_2 = lambda_1 v_1 + lambda_2v_2$$
          $$ iff$$
          $$ (lambda - lambda_1) v_1 + (lambda - lambda_2)v_2 = 0. $$
          Since $lambda_1 neq lambda_2$, $v_1$ and $v_2$ are linearly independent so
          $$ lambda - lambda_1 = 0 qquad lambda-lambda_2 = 0.$$
          So $ lambda = lambda_1 = lambda_2 $ which is a contradiction.






          share|cite|improve this answer












          By contradiction:



          If $v_1 + v_2$ is an eigenvector of A then there exists and eigenvalue $lambda$ so that $$ A(v_1 + v_2) = lambda(v_1 + v_2) = lambda v_1 + lambda v_2.$$
          However since $v_1$ and $v_2$ are eigenvectors and $A$ is linear we have
          $$ A(v_1 + v_2) = A(v_1) + A(v_2) = lambda_1 v_1 + lambda_2v_2.$$
          Therefore
          $$ lambda v_1 + lambda v_2 = lambda_1 v_1 + lambda_2v_2$$
          $$ iff$$
          $$ (lambda - lambda_1) v_1 + (lambda - lambda_2)v_2 = 0. $$
          Since $lambda_1 neq lambda_2$, $v_1$ and $v_2$ are linearly independent so
          $$ lambda - lambda_1 = 0 qquad lambda-lambda_2 = 0.$$
          So $ lambda = lambda_1 = lambda_2 $ which is a contradiction.







          share|cite|improve this answer












          share|cite|improve this answer



          share|cite|improve this answer










          answered 2 days ago









          Digitalis

          384114




          384114






















              up vote
              3
              down vote













              What you have above is true. It may help to write $lambda_2 = lambda_1 + c$ where $c neq 0$. Then you can write $A(v_1 + v_2) = lambda_1 (v_1 + v_2) + cv_2$. From here, you should be able to argue that $v_2$ is not parallel to $v_1 + v_2$, based on the assumption that $lambda_1 neq lambda_2$, so $v_1 + v_2$ cannot be an eigenvector of $A$.






              share|cite|improve this answer

























                up vote
                3
                down vote













                What you have above is true. It may help to write $lambda_2 = lambda_1 + c$ where $c neq 0$. Then you can write $A(v_1 + v_2) = lambda_1 (v_1 + v_2) + cv_2$. From here, you should be able to argue that $v_2$ is not parallel to $v_1 + v_2$, based on the assumption that $lambda_1 neq lambda_2$, so $v_1 + v_2$ cannot be an eigenvector of $A$.






                share|cite|improve this answer























                  up vote
                  3
                  down vote










                  up vote
                  3
                  down vote









                  What you have above is true. It may help to write $lambda_2 = lambda_1 + c$ where $c neq 0$. Then you can write $A(v_1 + v_2) = lambda_1 (v_1 + v_2) + cv_2$. From here, you should be able to argue that $v_2$ is not parallel to $v_1 + v_2$, based on the assumption that $lambda_1 neq lambda_2$, so $v_1 + v_2$ cannot be an eigenvector of $A$.






                  share|cite|improve this answer












                  What you have above is true. It may help to write $lambda_2 = lambda_1 + c$ where $c neq 0$. Then you can write $A(v_1 + v_2) = lambda_1 (v_1 + v_2) + cv_2$. From here, you should be able to argue that $v_2$ is not parallel to $v_1 + v_2$, based on the assumption that $lambda_1 neq lambda_2$, so $v_1 + v_2$ cannot be an eigenvector of $A$.







                  share|cite|improve this answer












                  share|cite|improve this answer



                  share|cite|improve this answer










                  answered 2 days ago









                  platty

                  2,093214




                  2,093214






















                      jake is a new contributor. Be nice, and check out our Code of Conduct.










                      draft saved

                      draft discarded


















                      jake is a new contributor. Be nice, and check out our Code of Conduct.













                      jake is a new contributor. Be nice, and check out our Code of Conduct.












                      jake is a new contributor. Be nice, and check out our Code of Conduct.
















                      Thanks for contributing an answer to Mathematics Stack Exchange!


                      • Please be sure to answer the question. Provide details and share your research!

                      But avoid



                      • Asking for help, clarification, or responding to other answers.

                      • Making statements based on opinion; back them up with references or personal experience.


                      Use MathJax to format equations. MathJax reference.


                      To learn more, see our tips on writing great answers.





                      Some of your past answers have not been well-received, and you're in danger of being blocked from answering.


                      Please pay close attention to the following guidance:


                      • Please be sure to answer the question. Provide details and share your research!

                      But avoid



                      • Asking for help, clarification, or responding to other answers.

                      • Making statements based on opinion; back them up with references or personal experience.


                      To learn more, see our tips on writing great answers.




                      draft saved


                      draft discarded














                      StackExchange.ready(
                      function () {
                      StackExchange.openid.initPostLogin('.new-post-login', 'https%3a%2f%2fmath.stackexchange.com%2fquestions%2f3016461%2fproving-v1v2-is-not-an-eigenvector-of-a%23new-answer', 'question_page');
                      }
                      );

                      Post as a guest















                      Required, but never shown





















































                      Required, but never shown














                      Required, but never shown












                      Required, but never shown







                      Required, but never shown

































                      Required, but never shown














                      Required, but never shown












                      Required, but never shown







                      Required, but never shown







                      Popular posts from this blog

                      Quarter-circle Tiles

                      build a pushdown automaton that recognizes the reverse language of a given pushdown automaton?

                      Mont Emei